LSAT and Law School Admissions Forum

Get expert LSAT preparation and law school admissions advice from PowerScore Test Preparation.

 Administrator
PowerScore Staff
  • PowerScore Staff
  • Posts: 8919
  • Joined: Feb 02, 2011
|
#24426
Complete Question Explanation

Strengthen-PR. The correct answer choice is (B)

To identify a principle that strengthens the argument, look for a broad rule that specifies what actions must be taken to achieve the goal outlined in the conclusion. Since the conclusion recommends that consumers buy books only from an independent bookstore and not from a bookstore chain, the correct principle must identify a rule applicable to consumers, not booksellers. This immediately eliminates answer choices (A) and (C).

The reason why the author recommends buying books from independent bookstores is that they tend not to reduce the variety of books available to consumers, unlike chain bookstores, which do. This argument makes a leap from fact to judgment: just because a retailer reduces the variety of products available to consumers, that does not mean that consumers should automatically shun those retailers. The missing rule should therefore establish a logical relationship between the consumers’ buying habits and the consequences of these habits on the variety of products available for purchase. Answer choice (B) is therefore correct.

Answer choice (A): This principle applies to chain bookstores, whereas the author’s recommendation is targetted at consumers. This answer choice fails to support the conclusion and is therefore incorrect.

Answer choice (B): This is the correct answer choice. See discussion above.

Answer choice (C): The interest of the bookselling business it not at issue in this argument. Furthermore, this answer choice states a fact, not what actions or judgments should apply in the given situation.

Answer choice (D): There is no evidence to suggest that chain bookstores deliberately force competing bookstores out of business. That the existence of independent bookstores is threatened by chain bookstores is not, by itself, suggestive of some deliberate motive or intention. This answer choice is incorrect.

Answer choice (E): What consumers should do if they have no access to any independent bookstore is outside the scope of this argument. This answer choice is incorrect.
 bonnie_a
  • Posts: 32
  • Joined: Jun 05, 2021
|
#90076
Hello, I have a question about D. I understand that it's wrong because chain bookstores' threatening the existence of independent bookstores might not necessarily be a "deliberate" act. Also, I was wondering if D could be wrong because "any" bookstore that deliberately forces competing bookstores out of business (as stated in D) may also include independent bookstores. Though the stimulus does not directly mention this about independent bookstores, I thought perhaps they could also be threatening or forcing others out of business. Since forcing other competing bookstores out of business sounds somewhat general, independent bookstores may have done it or are doing it now for themselves. Thank you!
 Robert Carroll
PowerScore Staff
  • PowerScore Staff
  • Posts: 1783
  • Joined: Dec 06, 2013
|
#90127
bonnie,

Either reason is a reason answer choice (D) is out, so that sounds good to me!

Robert Carroll

Get the most out of your LSAT Prep Plus subscription.

Analyze and track your performance with our Testing and Analytics Package.